This axiomatic system seems contradictory. Any thoughts?

  • Thread starter Thread starter jdinatale
  • Start date Start date
  • Tags Tags
    System Thoughts
Click For Summary
The discussion revolves around a contradiction found in a geometric axiomatic system. The user outlines a scenario with six points on line l_1 and introduces two additional points, G and H, leading to the conclusion that multiple lines can be drawn, including parallel lines that contradict a specified axiom. A response highlights that the perceived parallelism may not hold true within the axiomatic framework, suggesting that intersections in the diagram do not necessarily imply intersections in the axiomatic system. The need to prove that each line is incident with six points is emphasized as essential for resolving the contradiction. The conversation underscores the complexities and nuances of geometric axioms.
jdinatale
Messages
153
Reaction score
0

Homework Statement


axiomatic.jpg


edit: WHOOPS. Almost forgot to

The Attempt at a Solution



Ok. So I began with the line l_1 with exactly 6 points on it, A, B, C, D, E, F. (Axiom 2). Now, by axiom 1, there must exist 2 additional points not on this line. So I formed those, G and H.

No by Axiom 3, there exists a line between each of the points on L_1 and G and H, as well as a line between G and H. I drew those. Notice, however, that the line HF is parallel to GA, GB, GC, GD, and GE.

This contradicts axiom 4 which states that through G, only 1 line is parallel to HF!

axiomsystem.jpg
 
Last edited:
Physics news on Phys.org
Geeze, this forum doesn't seem to like geometry very much.
 
jdinatale said:

Homework Statement


axiomatic.jpg


edit: WHOOPS. Almost forgot to


The Attempt at a Solution



Ok. So I began with the line l_1 with exactly 6 points on it, A, B, C, D, E, F. (Axiom 2). Now, by axiom 1, there must exist 2 additional points not on this line. So I formed those, G and H.

No by Axiom 3, there exists a line between each of the points on L_1 and G and H, as well as a line between G and H. I drew those. Notice, however, that the line HF is parallel to GA, GB, GC, GD, and GE.

This contradicts axiom 4 which states that through G, only 1 line is parallel to HF!

axiomsystem.jpg

Only three of your lines show 6 points. Those are l1, p and, u. So, those lines you claim are parallel to line q (also known as HF) might actually intersect line q.
 
SammyS said:
Only three of your lines show 6 points. Those are l1, p and, u. So, those lines you claim are parallel to line q (also known as HF) might actually intersect line q.

Just because two lines "intersect" in the diagram, doesn't mean that they actually intersect within the axiom system. This model wouldn't work if that was the case because there would exist 7 points on AH (counting all of the intersections).

I have to PROVE that each line is incident with 6 points.
 
Question: A clock's minute hand has length 4 and its hour hand has length 3. What is the distance between the tips at the moment when it is increasing most rapidly?(Putnam Exam Question) Answer: Making assumption that both the hands moves at constant angular velocities, the answer is ## \sqrt{7} .## But don't you think this assumption is somewhat doubtful and wrong?

Similar threads

  • · Replies 36 ·
2
Replies
36
Views
6K
  • · Replies 1 ·
Replies
1
Views
3K
  • · Replies 13 ·
Replies
13
Views
3K
  • · Replies 7 ·
Replies
7
Views
3K
  • · Replies 46 ·
2
Replies
46
Views
8K
Replies
29
Views
5K
  • · Replies 2 ·
Replies
2
Views
3K
  • · Replies 2 ·
Replies
2
Views
2K
  • · Replies 11 ·
Replies
11
Views
3K
  • · Replies 51 ·
2
Replies
51
Views
5K